LSAT and Law School Admissions Forum

Get expert LSAT preparation and law school admissions advice from PowerScore Test Preparation.

 acp25
  • Posts: 13
  • Joined: Sep 21, 2017
|
#71510
Aloha Powerscore,

This game took me awhile to answer. I originally got 5/7 correct. I figured out why my answers were wrong for the ones I missed using the diagram below. But I'd like to know if I made the right inferences.

Game rules:
XX - where X stands for a variable that is repeated twice
J :arrow: HJ block
X is in 4 :arrow: X is not in 3
G :arrow: GO vertical block or GJ vertical block

I had two templates:

K G _ _
H J O _
1 2 3 4

K/ /K G _
H J O _
1 2 3 4

First, I made the first template where I figured that it must be that the HJ block can only occur where H is in 1 and J is in 2 and no where else because if J is present, it must be preceded by H, and no where else in the set up is able to reflect that.

Then I realized that there is a GO or GJ vertical blocks so I ended with two templates reflecting that.
Just realized that there could be three templates as the second template could be split.

Thank you in advance,
Amy
 Jeremy Press
PowerScore Staff
  • PowerScore Staff
  • Posts: 1000
  • Joined: Jun 12, 2017
|
#71532
Hi Amy,

You've got it exactly right! Those are the three templates I created for this game also. Templates are a really great way to approach the questions in this game, assuming you can spot them in the setup in a timely way and get the basic options sketched out quickly enough to leave time to answer the questions.

With your second template (that as you correctly note could be split into two options, depending on which week you place K into), I wouldn't add a thing.

In the first template, you could (though it's not entirely necessary) add into week 3 that one of L or M will be the product advertised with O. That is because H, K, J, and G have already been used in weeks 1 and 2, and therefore cannot be advertised a second time in week 3. The other member of the M/L pair will have to be advertised in week 4, because it won't have been advertised previously. The final product to be advertised in week 4 is either H or K (since G cannot go with M or L; and J cannot be displayed in week 4).

Nice job on that setup!

Jeremy
 kaileyahouck@gmail.com
  • Posts: 6
  • Joined: Mar 23, 2021
|
#87804
Hi,
I needed help walking through the set-up of rules for the first Killer Games in the Logic Games Lesson 11 HW and couldnt find an explanation. If someone could walk me through or point me to the forum where these games are discussed that would be great!
User avatar
 Stephanie Oswalt
PowerScore Staff
  • PowerScore Staff
  • Posts: 811
  • Joined: Jan 11, 2016
|
#87824
kaileyahouck@gmail.com wrote: Thu Jun 10, 2021 8:06 pm Hi,
I needed help walking through the set-up of rules for the first Killer Games in the Logic Games Lesson 11 HW and couldnt find an explanation. If someone could walk me through or point me to the forum where these games are discussed that would be great!
Hey Kailey!

I moved your question to the thread discussing this topic — you can find the full setup on page 1: viewtopic.php?f=353&t=8976.

Once you finish each Killer Game, you should be redirected to the Results Page in your Online Student Center. The question number on that Results Page is actually a link that will pop up and tell you the question text as well as the game it's sourced from. If you need additional assistance navigating that, email us at contact@powerscore.com and we can send more specific directions. :)

Thanks!
User avatar
 yenisey
  • Posts: 19
  • Joined: Oct 14, 2021
|
#92837
I made a mistake cause of rule 4 which says: "K is advertised during one of the first two weeks." I assumed K can't be in group 3 and 4. How the rule would be worded if K couldn't indeed be in group 3 and 4 ?
 Adam Tyson
PowerScore Staff
  • PowerScore Staff
  • Posts: 5153
  • Joined: Apr 14, 2011
|
#92846
There are a few ways they could have written that rule to mean that K was not allowed to appear in weeks 3 and 4, yenisey. Here are a few:

K is advertised only during one of the first two weeks

K is advertised no later than week 2

K is advertised exactly once, in either week 1 or else week 2

Remember that in all cases, if a rule does not prohibit something from happening, then you have to consider whether it could happen. That rule about K does not prohibit it from also being in week 4, so it could! But it cannot also appear in week 3 because of the portion of the second rule that prohibits a product from being advertised in week 3 and also in another week. That rule means that the duplicate (the product that goes twice) must appear in weeks 1 and 4 or else in weeks 2 and 4. K could do one of those!
User avatar
 yenisey
  • Posts: 19
  • Joined: Oct 14, 2021
|
#104524
I made a mistake in this game since I thought “K" could not be advertised during weeks 3 and 4 automatically since the fourth rule states: “K is advertised during one of the first two weeks.” I made the same mistake with the “O” since the last rule states: “O is one of the products advertised during week 3.” I thought this rule meant “O” can only be advertised during week 3.
In another game, it was: “Xena and exactly three other technicians repair radios.” Here I thought Xena could only repair the radio.
Is there any way I can be certain about the scope of the rules? How would they word the rule if it indeed meant “K” is only advertised during one of the first two weeks?
User avatar
 yenisey
  • Posts: 19
  • Joined: Oct 14, 2021
|
#104525
I see I already posted the same question. I got it. Thanks

Get the most out of your LSAT Prep Plus subscription.

Analyze and track your performance with our Testing and Analytics Package.